¿Cuál es la conclusión del efecto Aharonov-Bohm?

¿Cuál es la conclusión que podemos sacar del efecto Aharonov-Bohm ? ¿Simplemente sugiere que el vector potencial tiene efectos medibles? ¿Significa que es un observable real en mecánica cuántica? ¿Este efecto tiene algo que ver con la topología del espacio ?

Se debe tener en cuenta que la controversia continúa (ver también esta pregunta ) y que la física en su conjunto todavía tiene dificultades para extraer una única "conclusión" clara sobre este efecto.

Respuestas (2)

Más allá de las polémicas, creo que todavía se puede responder a esta pregunta.

El potencial de calibre no puede ser un observable en mecánica cuántica, ya que es covariante de calibre. La mecánica cuántica es clara al respecto. En una primera versión cuantificada de QM, solo mide | Ψ | 2 , y no se ve afectado por una transformación de indicador.

Este párrafo realmente no responde a su pregunta, pero puede decirle algo interesante. Nótese que en el experimento de la doble rendija el observable es el módulo al cuadrado del campo eléctrico en una posición dada. Así que algo que es calibre invariante . A continuación, si observa solo una posición (por ejemplo, en la pantalla), no debería notar nada : tiene algo de luz que llega a la pantalla. ¡Multa! Necesita mirar diferentes posiciones para registrar un cambio de intensidad. Dado que la mayoría de las veces ves la pantalla completa cuando realizas el experimento, el comentario anterior apenas se hace. Para el efecto Aharonov-Bohm, registra una corriente, no el potencial del indicador. De esta corriente infieres que algo se pone extrañocuando cambia el campo magnético . Obviamente, el patrón de interferencia depende del flujo magnético dentro del bucle. Entonces, para un valor de flujo, obtienes una ruta y ninguna interferencia. Dado que la corriente es invariable según el indicador de construcción, todos están contentos.

Por supuesto, la topología importa, ya que es un efecto magnético. Existen efectos similares a Aharonov-Bohm más recientes, donde lo que importa es la topología del propio espacio de Hilbert, no la topología en el espacio. Véase, por ejemplo, fase Berry en Wikipedia. Lo siento, el tema ahora es tan amplio que prefiero no entrar en detalles.

En cuanto a la interpretación del efecto AB, es una vez más una cuestión de conveniencia: las matemáticas gobiernan, ¡las palabras no!

Sin embargo, un hecho extraño: ¿por qué registramos algo así como un flujo magnético cuando no hay campo magnético? Bueno, porque un flujo no es local, ya que uno necesita cerrar el ciclo para observarlo. Ese es el efecto realmente extraño que perjudica a la mayoría de los físicos nacidos después de Einstein, porque realmente parece no local. Pero nunca entendí realmente por qué, ya que un flujo era un flujo antes que él, y nunca discutió la materia cargada (en oposición a las partículas cargadas , como en su artículo de 1905). Creo que la mayoría de las polémicas provienen de la sutil diferencia entre un campo de calibre, un potencial de calibre y las integrales de ellos, que son iguales gracias al teorema de Stoke [ver, por ejemplo , ¿Podemos medir un campo electromagnético?] Además, la no localidad (a pesar de que aún se debate) está bien establecida para la mayoría de los físicos.

A continuación, este hecho aún más extraño: ¿por qué necesitamos tomar el módulo cuadrado de la suma de las diferentes amplitudes? | i Ψ i | 2 , y no la suma del módulo cuadrado de la amplitud i | Ψ i | 2 ? Esta sería una pregunta sobre el principio de Huygens (o sobre las integrales de ruta de Feynman si desea ser pedante), no sobre el efecto AB. Aún así, el efecto AB es una hermosa demostración del principio de Huygens, como el experimento de la doble rendija: el flujo electromagnético dentro del bucle hace que los dos caminos tengan longitudes diferentes.

El efecto AB es, pues, un

  • demostración clara del principio de calibre
  • demostración clara del principio de Huygens
  • demostración clara de que estos dos principios juntos dan efectos observables

Para estar seguro: lo que llamé un principio de calibre aquí es el hecho de que una transformación de calibre A A + x obliga a transformar la función de onda también Ψ mi i x Ψ .

El debate sobre el potencial de calibre frente al campo de calibre claramente no tiene sentido, ya que lo que registra se puede expresar en términos de ambos.

El debate sobre la no localidad es mucho más sutil, y debe advertirlo cuando aprenda el efecto Aharonov-Bohm o la (pseudo-)paradoja de Einstein-Podolsky-Rosen.

Esta declaración es extraña: "El potencial de calibre no puede ser observable en la mecánica cuántica, ya que es covariante de calibre. La mecánica cuántica es clara al respecto". No es cierto que la mecánica cuántica implique invariancia de calibre.
@ my2cts Seguro que tienes razón. Es por eso que agrego la siguiente oración: "El potencial de calibre no puede ser un observable en la mecánica cuántica, ya que es covariante de calibre. La mecánica cuántica es clara al respecto. En una primera versión cuantizada de QM, mide solo |Ψ|2 , y no se ve afectado por una transformación de calibre". Entonces, para responder a su comentario: QM no implica invariancia de calibre, pero QM no permite que la cantidad covariante de calibre sea medible. Entonces, el potencial de calibre no puede ser observable en la mecánica cuántica.
La ecuación de Schrödinger generalmente da resultados invariantes de calibre debido al llamado acoplamiento mínimo, pero esto no implica invariancia de calibre.
Eliminé algunos comentarios y respuestas levemente inapropiados. Tenga en cuenta nuestras expectativas de comportamiento en este sitio.

El efecto AB es invariante de calibre. Al mismo tiempo, muestra que la invariancia de calibre no implica que el potencial no sea físico.